x, y, z

Функция Римана непрерывна во всех иррациональных и разрывна во всех рациональных точках

# 22 Июл 2016 21:15:55
Evgeniy

Эта функция имеет и много других названий: функция Томе (примеч. Carl Johannes Thomae (1840 – 1921) — немецкий математик), модифицированная функция Дирихле, поп-корн (popcorn) функция, функция дождевых капель (raindrop), функция счетных облаков (countable cloud), функция линейки (ruler) или Звезды над Вавилоном (Stars over Babylon).

Функция Римана является простейшим примером функции, которая непрерывна во всех иррациональных точках и разрывна во всех рациональных точках. Функция Римана определяется так:

$f(x)= \begin{cases} \dfrac{1}{n}, & \text{если}\ x=\dfrac{m}{n},\ m\in\mathbb{Z},\, n\in\mathbb{N}\\ 0, & \text{если}\ x\in\mathbb{R}\setminus\mathbb{Q}, \end{cases}$

где $\dfrac{m}{n}$ — несократимая дробь (для любого рационального числа существует его представление в виде несократимой дроби).

График функции Римана

Докажем, что функция Римана непрерывна во всех иррациональных точках.

Для произвольного $\varepsilon > 0$ рассмотрим множество

$M=\{x\in\mathbb{R}\colon f(x) \ge \varepsilon\}$.

Никакая иррациональная точка не лежит в $M$, поскольку в иррациональных точках функция $f$ обращается в ноль.

Если $x\in M$, тогда $x$ есть рациональное число вида $x=\dfrac{m}{n}$, где $m\in\mathbb{Z},\ n\in\mathbb{N}$, дробь $\dfrac{m}{n}$ несократима, и тогда $f(x)=\dfrac{1}{n} \ge \varepsilon$ и, следовательно, $n \le \dfrac{1}{\varepsilon}$. Из ограничения на $n$ следует, что пересечение множества $M$ и любого ограниченного интервала состоит из конечного числа точек.

Пусть $\alpha$ — произвольное иррациональное число. По определению $f(\alpha)=0$. Мы можем выбрать окрестность точки $\alpha$ так, чтобы в ней не содержалась ни одна точка множества $M$. Если же $x \notin M$, то $f(x) < \varepsilon$. Таким образом, мы нашли интервал, который требуется в определении непрерывности.

Теперь докажем, что функция Римана разрывна во всех рациональных точках.

Пусть $q$ — произвольное рациональное число. По определению $f(q)>0$. В любой окрестности рационального числа $q$ найдутся иррациональное число $\beta$ и $f(\beta)=0$. Таким образом, условие непрерывности не выполняется.
# 6 Янв 2020 17:50:59
ИгорьМехМат

Единственный коммент, на такое классное объяснение!

Вижу так мало комментариев(ни одного), поэтому решил написать свой о том как хороша эта информация, спасибо вам большое понял очень точно что и хотел, уже не зря потратили своё время, благодарю!
# 20 Окт 2022 22:52:59
Андрей МФТИ

Отличная заметка!

Автору - респект, кратко и понятно разложил всё по полочкам. Спасибо!
*Имя:
Заголовок:
[tex-clear] [tex-help] [ted]
  • formulas >

* Сколько символов на картинке?
Captcha
Отправляя данные, вы соглашаетесь с Правилами сайта.